66
125) A patient with Bloom Syndrome is most likely to have which laboratory abnormalities: A. Elevated IgE B. Positive ANA C. Thrombocytopenia D. Macrocytic anemia E. Decreased immunoglobulinsCorrect Choice Bloom syndrome is an autosomal recessive disorder due to a mutation in the BLM gene which codes for a DNA helicase. Patients have impaired DNA repair after UV exposure and increased photosensitivity. Clinical features include photodistributed erythema, cheilitis, high-pitched voice, hypogonadism, and increased risk for leukemia, lymphoma and GI adenocarcinoma. Laboratory evaluation reveals decreased IgA, IgM and IgG leading to increased risk of respiratory infections 126) A patient with hypohydrosis and hyperpyrexia, anodontia, and sparse hair has which syndrome: A. Dyskeratosis congenita B. Papillon Lefevre C. Pachyonychia congenita D. Hidrotic ectodermal dysplasia E. Anhidrotic ectodermal dysplasiaCorrect Choice Anhidrotic ectodermal dysplasia is an x-linked recessive disorder which presents with the triad of hypohydrosis (or anhidrosis) with hyperpyrexia, anodontia (other dental findings include peg teeth, molars with hooked cusps) and sparse hair. Patients tend to overheat 127) A mentally retarded patient is found to also have a malar rash and downward lens displacement. Another clinical feature might include: A. periodontitis B. hypertrichosis C. deep venous thrombosesCorrect Choice 47

ETAS_MCQ_03 b genodermatoses

Embed Size (px)

Citation preview

Page 1: ETAS_MCQ_03 b genodermatoses

125) A patient with Bloom Syndrome is most likely to have which laboratory abnormalities:

A. Elevated IgE

B. Positive ANA

C. Thrombocytopenia

D. Macrocytic anemia

E. Decreased immunoglobulinsCorrect Choice

Bloom syndrome is an autosomal recessive disorder due to a mutation in the BLM gene which codes for a DNA helicase. Patients have impaired DNA repair after UV exposure and increased photosensitivity. Clinical features include photodistributed erythema, cheilitis, high-pitched voice, hypogonadism, and increased risk for leukemia, lymphoma and GI adenocarcinoma. Laboratory evaluation reveals decreased IgA, IgM and IgG leading to increased risk of respiratory infections

126) A patient with hypohydrosis and hyperpyrexia, anodontia, and sparse hair has which syndrome:

A. Dyskeratosis congenita

B. Papillon Lefevre

C. Pachyonychia congenita

D. Hidrotic ectodermal dysplasia

E. Anhidrotic ectodermal dysplasiaCorrect Choice

Anhidrotic ectodermal dysplasia is an x-linked recessive disorder which presents with the triad of hypohydrosis (or anhidrosis) with hyperpyrexia, anodontia (other dental findings include peg teeth, molars with hooked cusps) and sparse hair. Patients tend to overheat

127) A mentally retarded patient is found to also have a malar rash and downward lens displacement. Another clinical feature might include:

A. periodontitis

B. hypertrichosis

C. deep venous thrombosesCorrect Choice

D. multiple nevi

E. short stature

The above described patient has homocystinuria, an autosomal recessive disorder caused by a deficiency in cystathionine beta-synthase, leading to an accumulation of homocysteine. Clinical

47

Page 2: ETAS_MCQ_03 b genodermatoses

features include: malar flush, DVT’s, livedo reticularis, leg ulcers, downward lens displacement, myopia, glaucoma, sparse fine hair, mental retardation, and musculoskeletal anomalies

128) Which type of epidermolysis bullosa is associated with mitten deformities of the hands?

A. Generalized atrophic benign epidermolysis bullosa (GABEB)

B. Dominant dystrophic

C. Weber-Cockayne

D. Recessive dystrophic Correct Choice

E. Herlitz type

The recessive dystrophic type of epidermolysis bullosa is associated with chronic hand ulcers resulting in scarring that causes mitten deformities. These chronic scars often result in the formation of fatal squamous cell carcinomas.

129) Nevoid basal carcinoma syndrome (Gorlin syndrome) is autosomal dominant transmitted mutation of the patched gene. Symptoms include innumberable basal cell carcinomas, painful odontogenic jaw keratocysts, palmoplantar pits, frontal bossing, bifid ribs and what other bony abnormality?

A. Osteopoikilosis

B. Polyostotic fibrous dysplasia

C. Calcification of falx cerebriCorrect Choice

D. Sphenoid wing dysplasia

E. Stippled epiphyses

Calcification of falx cerebri is seen in Gorlin's syndrome. CHILD syndrome and chondrodysplasia punctata both can exhibit stippled epiphyses. Polyostotic fibrous dysplasia is found in McCune-Albright syndrome, osteopoikilosis in seen in Buschke-Ollendorf syndrome. Sphenoid wing dysplasia is seen in neurofibromatosis type I

130) A double row of eyelashes is associated with:

A. Lymphedema-distichiasis syndromeCorrect Choice

B. Hunters syndrome

C. Cornelia de Lange syndrome

D. Rubinstein-Taybi syndrome

E. Russell-Silver syndrome

48

Page 3: ETAS_MCQ_03 b genodermatoses

A double row of eyelashes is defined as distichiasis and is associated with the Lymphedema-distichiasis syndrome. This syndrome is transmitted in an autosomal dominant fashion and is related to a mutation in FOXC2. Findings include late onset lymphedema, distichiasis, corneal irritation, ectropion, webbed neck and congenital heart defects. The remaining syndromes do not include distichiasis as a feature.

131) Patients with Russell-Silver syndrome exhibit:

A. Broad thumbs

B. Clinodactyly of fifth fingerCorrect Choice

C. Peg teeth

D. Shortened 4th and 5th metacarpals

E. Osteopathia striata

Characteristic features of Russell-Silver include short stature, bony asymmetry, triangular facies, clinodactyly of fifth finger, and precocious sexual development with cryptochordism/hypospadias. Broad thumbs are seen in Rubinstein-Taybi and shortened fourth and fifth metacarpals are seen in Turner syndrome. Osteopathia striata is characteristic of focal dermal hypoplasia

132) A child presents with macroglossia, exopthalmos and gigantism. He has a history of omphalocele repair and has circular depressions on the rim of the posterior helices. Although this syndrome is most often transmitted in a sporadic manner, 15% of cases have defects in which gene?

A. FLT4

B. VHL

C. FOXC2

D. ATM

E. KIP2Correct Choice

KIP2 can be mutated in 15% of cases of Beckwith-Wiedemann Syndrome. KIP2 is a cyclin-dependent kinase inhibitor gene which acts as a negative regulator of cell proliferation. These children have an increase risk of Wilms' tumor and organomegaly. In addition to the circular ear depression, they can also have a linear earlobe crease. The remaining genes are mutated in other syndromes with vascular disorders as a component: ATM in ataxia telangectasia, VHL in Von Hippel-Lindau, FLT4 in Hereditary lymphedema and FOXC2 in Lymphedema-distichiasis syndrome

133) Which of the following is defective in Ehlers-Danlos syndrome (EDS) with congenital adrenal hyperplasia?

A. Lysyl hydroxylase

B. Tenascin-XCorrect Choice

C. Lysyl oxidase

49

Page 4: ETAS_MCQ_03 b genodermatoses

D. All of these answers are correct

E. None of these answers are correct

Tenascin-X defects are associated with EDS and with congenital adrenal hyperplasia. The phenotype is that of typical EDS with hyperextensible skin, hypermobile joints, and tissue fragility. Lysyl oxidase is defective in X-linked EDS (type V) and Occipital horn syndrome (type IX). Lysyl hydroxylase is defective in ocular-scoliotic (type VI) EDS

134) What is the most likely gene mutation in this individual who has migratory patches and fixed plaques as depicted in this picture?

A. Connexin 26

B. Connexin 30.3 and 31Correct Choice

C. Keratins 1 and 10

D. Calcium ATPase 2C1

E. Loricrin

Erythrokeratodermis variabilis is an autosomal dominant disease characterized by transient patches of geographic erythema and fixed hyperkeratotic plaques. The disease is due to gene defects in connexin 30.3 and 31

135) The gene that is responsible for this disease also plays a pathogenic role in what other disorder?

A. MastocytosisCorrect Choice

B. Sarcoidosis

C. Hereditary Angioedema

D. Mycosis Fungoides

E. Waardenburg Syndrome

Piebaldism is an autosomal dominant disease characterized by depigmented patches and a white forelock. It is caused by a mutation in the c-kit proto-oncogene. Mutations in c-kit have also been found in the peripheral cells of individuals with mastocytosis

136) Menkes kinky hair syndrome is associated most commonly with which of the following hair abnormalities?

A. Pili triangulati et canaliculati

B. Pili tortiCorrect Choice

50

Page 5: ETAS_MCQ_03 b genodermatoses

C. None of these options are correct

D. Trichorrhexis invaginata

E. Trichorrhexis nodosa

While pili torti is not exclusively found in Menkes kinky hair syndrome, this is the most common hair abnormality found. Trichorrhexis nodosa can be also seen. Trichorrhexis invaginata is commonly found in Netherton syndrome. Pili triangulati et canaliculati is the finding seen in Uncombable hair syndrome.

137) Medulloblastomas are seen in which of the following syndromes?

A. Familial cylindromatosis

B. Nevoid basal cell carcinomas syndrome Correct Choice

C. Bazex syndrome

D. Nicolau-Balus syndrome

E. Birt-Hogg-Dube syndrome

Nevoid basal cell carcinomas syndrome (Gorlin syndrome) is an autosomal dominant condition caused by mutations in the patched gene, which is involved in the hedgehog signaling pathway. Patients develop innumerable BCCs, palmoplantar pits, painful odontogenic jaw keratocysts, frontal bossing, bifid ribs, and calcification of the falx cerebri. Medulloblastomas and ovarian fibromas and fibrosarcomas are associated with this condition

138) Which of the following is caused by a defect in a gap junction protein?

A. Hailey-hailey

B. Epidermolysis bullosa simplex

C. Bullous ichthyosis of siemens

D. Erythrokeratoderma variabilisCorrect Choice

E. Dyskeratosis congenita

Erythrokeratoderma variabilis is also known as Mendes da Costa Syndrome. It is caused by a defect in connexin 31, a gap junction protein. EB simplex is caused by a mutation in keratins 5 & 14, Hailey-Hailey is caused by a mutation in calcium transporters, dyskeratosis congenita is caused by a defect in rRNA synthesis, and bullous icthyosis of Siemens is caused by mutations in keratin 2e

139) Which of the following syndromes is associated with markedly increased IgE levels, cold abscesses and a characteristic coarse facies?

A. Chronic granulomatous disease

51

Page 6: ETAS_MCQ_03 b genodermatoses

B. Wiskott-Aldrich syndrome

C. Job syndromeCorrect Choice

D. Leiner's disease

E. Severe combined immunodeficiency

Job syndrome or Hyper IgE syndrome is characterized by these findings. In addition, there is a peripheral eosinophilia, eczematous dermatitis, frequent bronchitis and pneumonia, otitis media and sinusitis. The other listed conditions are associated with immunodeficiency. Wiskott-Aldrich can have eczematous dermatitis and all of these syndromes will have abnormal infections. They do not have markedly increased levels of IgE like Job syndrome.

140) Pruritus is Sjogren Larsson syndrome is attributed to accumulation of what molecule(s)?

A. LeukotrieneCorrect Choice

B. Histamine

C. Bile salts

D. None of these answers are correct

E. All of these answers are correct

Accumulation of leukotriene B4 contributes to pruritus in Sjogren Larsson syndrome. Leukotriene inhibitors may be helpful in controlling symptoms.

141) Which of the following syndromes is associated with tricholemmomas?

A. Rasmussen's

B. Bannayan-Riley-RuvacalbaCorrect Choice

C. Basex

D. Birt-Hogg-Dube

E. Brooke-Spiegler

Tricholemmomas are seen in Bannayan-Riley-Ruvacalba syndrome. This is an autosomal dominant condition with macrocephaly, lipomas, hemangiomas, skeletal abnormalities, lymphangioma circupscriptum, angiokeratomas, penile lentigines, acanthosis nigricans, and achrocrodons. There is an increased incidence of breast, thyroid, and GI cancers. Tricholemmomas are also seen in Cowden syndrome

142) Electron microscopic examination of a hair shaft reveals a canal-like groove along the shaft of a triangular-shaped hair. This patient has:

52

Page 7: ETAS_MCQ_03 b genodermatoses

A. Menke’s Kinky Hair syndrome

B. Spun-glass hairCorrect Choice

C. Trichothiodystrophy

D. Netherton’s syndrome

E. Bjornstad syndrome

Pili trianguli et canaliculi is also known as Spun-glass hair or Uncombable Hair Syndrome. Netherton patients have trichorexis invaginata, Menke’s kinky hair patients have short, brittle sparse hairs, “tiger tail” hair is seen in trichothiodystrophy, and pili torti is seen in bjornstad syndrome

143) A patient with coarse facies, broad nasal bridge, and extensive eczema might be expected to have which abnormal laboratory value?

A. Hypertriglyceridemia

B. High Copper levels

C. High IgE levelsCorrect Choice

D. Low Zinc levels

E. Anemia

Patients’ with Hyper IgE Syndrome (Job syndrome) have impaired regulation of the IgE function and increased susceptibility to infections. In addition to recurrent cutaneous infections including cold abscesses, patients have widespread eczematous dermatitis, recurrent sinopulmonary infections and typically have coarse facies with broad nasal bridge and a prominent nose

144) Reticulate pigmentation of skin, poikiloderma, alopecia, nail atrophy, premalignant oral leukoplakia, and a Fanconi-type pancytopenia resulting in early death in addition to posterior fossa malformations is characteristic of which of the following syndromes

A. Cockayne syndrome

B. Hoyeraal-Hreidarsson syndromeCorrect Choice

C. Bloom syndrome

D. Dyskeratosis congenita

E. Wiskott-Aldrich syndrome

Hoyeraal-Hreidarsson syndrome is has all of the features of dyskeratosis congenita plus posterior fossa malformations. Bloom syndrome and Cockayne syndrome both have poikiloderma as features, but do not include posterior fossa malformations as part of the syndrome. Wiskott-Aldrich syndrome does not include any of these findings

145) Dyskeratosis congenita has all of the findings listed below EXCEPT:

53

Page 8: ETAS_MCQ_03 b genodermatoses

A. reticulate pigmentation of skin/poikiloderma

B. alopecia

C. Mental retardationCorrect Choice

D. Is allelic to Hoyerall-Hreidarsson syndrome

E. Fanconi-type pancytopenia resulting in early death

Mental retardation is not part of the spectrum of disease of dyskeratosis congenita. The remaining options are correct. Hoyerall-Hreidarsson syndrome has all of the findings of dyskeratosis congenita with the addition of posterior fossa malformations

146) In ataxia telangectasia, the ATM gene is mutated. The product of the ATM gene is an enzyme which:

A. participates in NF-kB activation

B. binds transforming growth factor beta protein

C. responds to DNA damage by phosphorylating key DNA repair substratesCorrect Choice

D. is the VEGF receptor 3

E. is an inhibitor of G1 cyclin/Cdk complexes

The ATM gene is a member of the phosphatidylinositol-3 family of proteins that respond to DNA damage by phosphorylating key substrates involved in DNA repair according to OMIM. Defects in endoglin (TGF beta3 binding protein) is deficient in Osler-Weber-Rendu syndrome. The VEGF receptor 3 is defective in hereditary lymphedema. The NEMO gene is defective in Incontinentia Pigmenti. Its product, NF-kB essential modulator (NEMO) is a key activator in the NF-kB pathway. KIP2 is involved in AD Beckwith-Wiedemann syndrome and is an inhibitor of G1 cyclin/Cdk complexes

147) Ichthyosis hystrix is characterized by the following gene defects?

A. keratins 5 and 14

B. keratins 1 and 10Correct Choice

C. keratins 1 and 9

D. none of these answers are correct

E. keratins 6 and 16

Ichthyosis hystrix or extensive epidermal nevi occurs secondary to a somatic mosaicism for keratins 1 and 10. If the mosaicism occurs on gonadal cells, offspring may have full blown epidermolytic hyperkeratosis (EHK).

148) Mucosal malignancy is a complication of:

54

Page 9: ETAS_MCQ_03 b genodermatoses

A. Oral hairy leukoplakia

B. Chronic candidiasis

C. Focal epithelial hyperplasia

D. White sponge nevus

E. Dyskeratosis congenitalCorrect Choice

Dyskeratosis congenital, also called Zinsser-Engman-Cole syndrome, is an X-linked recessive genodermatosis caused by a mutation in DKC1. DKC1 encodes for dyskerin, which helps to maintain telomeres through the pseudouridylation of rRNA. Features of this condition include reticulate gray-brown hyperpigmentation, dystrophic nails, alopecia and Fanconi’s type pancytopenia. Patients may have premalignant leukoplakia which should be followed closely

149) What condition is associated with this finding of inflammatory keratotic facial papules which may result in scarring and atrophy?

A. Reiter's syndrome

B. Chloracne

C. Keratosis pilaris

D. Ulerythema ophryogenesCorrect Choice

E. Systemic lupus erythematosus

Ulerythema ophryogenes is a rare disorder that affects children and young adults. It is characterized by keratosis pilaris atrophicans and loss of lateral third of eyebrow

150) Ichthyosis linearis circumflexa is one of the findings seen in the syndrome caused by which of the following genes?

A. SLURP1

B. CYLD

C. GJB2

D. LMX1B

E. SPINK5Correct Choice

The SPINK5 gene encodes for LEKTI, a serine protease inhibitor important in the regulation of proteolysis in epithelia formation and keratinocyte terminal differentiation, is mutated in Netherton’s Syndrome. Other findings include: trichorrhexis invaginata (bamboo, ball and socket hair), atopic dermatitis, and anaphylaxis from food allergy. SLURP1 is mutated in Mal de Maleda, GJB2 in Vohwinkel’s syndrome, LMX1B in Nail-Patella syndrome, and CYLD in Familial Cylindromiasis

151) What is the mode of transmission for lamellar ichthyosis?

55

Page 10: ETAS_MCQ_03 b genodermatoses

A. sporadic

B. X-linked dominant

C. autosomal dominant

D. autosomal recessiveCorrect Choice

E. X-linked recessive

Lamellar ichthyosis which is characterized by collodian membrane in newborns and platelike scale in children and adults is an autosomal recessive syndrome. The gene defect is transglutaminase 1 (TGM1

152) Which of the following diseases is caused by a mutation in a gap junction protein?

A. Schopf-Schulz-Passarge syndrome

B. Vohwinkel syndrome (classic) Correct Choice

C. Striated PPK

D. Mal de Meleda

E. Vohwinkel syndrome (ichthyotic)

Classic Vohwinkel syndrome is caused by mutations in connexin 26, a gap junction protein. Ichthyotic Vohwinkel syndrome is caused by mutations in loricrin and has ichthyosis but not deafness

153) Which of the following are cutaneous features of Marfan syndrome?

A. Dermatofibrosis lenticularis and striae

B. Fat herniation and cutaneous atrophy

C. Striae and elastosis perforans serpiginosa Correct Choice

D. Loose skin and crumpled ears

E. Sclerodermoid changes and dyspigmentation

Marfan syndrome is an autosomal dominant disorder caused by mutations in fibrillin 1 and 2. Patients have tall stature, arachnodactyly, pectus excavatum, high-arched palate, joint laxity, ectopia lentis with upward dislocation, dilated aorta with rupture, mitral valve prolapse, striae, and elastosis perforans serpiginosa

154) What gene defect would you expect to find in a child with white forelock, dystopia canthorum, and upper limb abnormalities?

56

Page 11: ETAS_MCQ_03 b genodermatoses

A. MITF

B. Endothelin-3

C. Pax3Correct Choice

D. SOX10

E. C-kit proto-oncogene

Waardenburg's syndrome is characterized by depigmented patches, white forelock, and deafness. Both type 1 and 3 are caused by mutations in Pax3. Type 3 is also associated with limb abnormalities. MITF and SOX10 defects are responsible for types 2 and 4 respectively. C-kit proto-oncogene mutatios are seen in piebaldism

155) Palmoplantar keratoderma with deafness is caused by a defect in which gene?

A. unknown

B. Plakophilin

C. lysosomal papain like cysteine proteinase

D. SLURP-1

E. mitochondrial serine transferase RNACorrect Choice

Palmoplantar keratoderma with deafness is caused by a defect in mitochondrial serine transferase RNA. A defect in SLURP-1 causes Mal de Meleda. A defect in plakophilin causes ectodermal dysplasia with skin fragility. A defect in cathepsin C lysososomal papain like cysteine proteinase causes Papillon LeFevre and Haim Munk.

156) A Puerto Rican woman is seen in clinic for a pruritic rash on her trunk. A punch biopsy is performed. The biopsy site continues to bleed, with hematoma formation. The bleeding is eventually controlled. On further exam, her skin and hair are light brown. She has a history of granulomatous colitis. What it the most likely reason she had excess bleeding with a simple procedure?

A. Her platelets lack dense bodies, causing excess bleeding Correct Choice

B. Her Factor VIII levels are low

C. Her intrinsic factor is deficient

D. She is congentially deficient in platelets

E. None of the answers are correct

Platelets without dense bodies cause excess bleeding in Hermansky-Pudlak syndrome. Other features of this condition include oculocutaneous albinism, ceroid lysosomal storage disease resulting in pulmonary fibrosis, granulomatous colitis

157) Which of the following syndromes are linked to a PTEN gene mutation?

57

Page 12: ETAS_MCQ_03 b genodermatoses

A. Cowden

B. All of these options are correctCorrect Choice

C. Lhermitte-Duclos disease

D. Banayan-Zonana

E. Proteus

PTEN gene produces a phosphatase the regulates the cell-cycle and apoptosis, therefore acting as a tumor suppressor gene. Tissues affected by this mutation are those with increased proliferation such as epidermis, the oral and gastrointestinal mucosa, and the thyroid and breast epithelium. All of the syndromes listed have mutations in PTEN.

158) The combination of painful palmoplantar keratoderma and pseudoherpetic keratitis is characteristic of which of the following syndromes?

A. Naxos syndrome

B. Howel-Evans syndrome

C. Richner-Hanhart syndrome Correct Choice

D. Schopf-Schulz-Passarge syndrome

E. Vohwinkel syndrome

Richner-Hanhart syndrome (tyrosenemia type II) is an autosomal recessive disorder caused by a deficiency in hepatic tyrosine aminotransferase. This disease is characterized by painful PPK, pseudoherpetic keratitis and blindness. Treatment is low-tyrosine/phenylalanine diet

159) Epidermal nevus syndromes inheritance pattern is:

A. Autosomal dominant

B. X-linked dominant

C. X-linked recessive

D. Autosomal recessive

E. sporadicCorrect Choice

Epidermal nevus syndrome has many findings, including: sporadic inheritance, nevus unius lateris, capillary malformations, café au lait macules, mantal retardation and seizures, deafness, hemiparesis, hemihypertrophy of limbs, kyphoscoliosis and rare solid tumors. A biopsy is helpful to rule out epidermolytic hyperkeratosis. If positive, the patient’s offspring are at risk for generalized epidermolytic hyperkeratosis

160) Ichthyosis bullosa of Siemens is a condition characterized by fragile blisters at birth, hyperkeratotic plaques on elbows/knees later in life, and a gene mutation in:

58

Page 13: ETAS_MCQ_03 b genodermatoses

A. Keratin 1/10

B. Keratin 6a/16

C. None of these options are correct

D. Keratin 5/14

E. Keratin 2eCorrect Choice

Keratin 2e is mutated in ichythosis bullosa of Siemens. Keratin 5 and 14 are defective in epidermolysis bullosa simplex, 6a/16 in Pachyonychia congenital type I (Jadassohn-Lewandowsky), 1/10 in epidermolytic hyperkeratosis and Unna-Thost PPK

161) Spastic ditetraplegia is associated with which of the following disorders?

A. Sjogren-Larsson syndrome Correct Choice

B. X-linked ichthyosis

C. Lamellar ichthyosis

D. Refsum syndrome

E. KID syndrome

Sjogren-Larsson syndrome is an autosomal recessive disorder caused by mutations in the fatty aldehyde oxidoreductase/alcohol dehydrogenase gene. This disorder is characterized by ichthyosis, spastic ditetraplegia, mental retardation, epilepsy, glistening dot retinal pigmentation, and dental enamel dysplasia

162) A sporadic syndrome affecting transcriptional coactivator CREB-binding protein is:

A. Nonne-Milroy disease

B. Rubinstein-Taybi syndromeCorrect Choice

C. Cornelia de Lange syndrome

D. Maffucci syndrome

E. Blue rubber bleb nevus syndrome

Rubinstein-Taybi syndrome is caused by a sporadically transmitted defect in transcriptional coactivator CREB-binding protein. This gene is responsible for encoding a nuclear protein which acts as a co-activator of cAMP regulated gene expression. Findings of this syndrome include: capillary malformation, short stature, broad thumbs, craniofacial abnormalities including beaked nose, mental retardation, strabismus, congenital heart defects and cryptorchidism. The other listed conditions are not related to this defect

163) The arylsulfatase C gene is mutated in which disease?

59

Page 14: ETAS_MCQ_03 b genodermatoses

A. Naxos syndrome

B. X-linked ichthyosisCorrect Choice

C. Griscelli syndrome

D. Haim-Munk syndrome

E. Refsum syndrome

Arylsulfatase C is also known as steroid sulfatase and is mutated in X-linked ichthyosis. This condition is inherited in a X-linked recessive pattern. Clinical findings include: brown scale sparing palms, soles and flexures, comma-shaped corneal opacities, failure of labor progression and cryptorchidism. It is also mutated in X-linked recessive type chondrodysplasia punctata

164) In one variant of epidermolysis bullosa simplex, those affected have muscular dystrophy in addition to the skin findings. Which protein mutation has been linked to this finding?

A. plectinCorrect Choice

B. plakophilin

C. plakoglobin

D. desmocollin

E. desmoglein

A plectin mutation is linked with this subtype of epidermolysis bullosa simplex.

165) A 3 year old boy presents with the findings seen in the image. He also has thrombocytopenia with purpura and a history of recurrent pyogenic bacterial infections. What is the most likely diagnosis in this child?

A. Hyper-IgE syndrome

B. Severe combined immunodeficiency

C. Wiskott-Aldrich syndromeCorrect Choice

D. Leiner syndrome

E. Chronic Granulomatous disease

The findings described are consistent with Wiskott-Aldrich syndrome. The characteristic triad can be simplified to the 3 P's - Pruritus (atopic dermatitis), Purpura (thrombocytopenia leading to purpura and other bleeding) and pyogenic infections. The remaining options are related to Wiskott-Aldrich in that they all have immunodeficiency as a feature, but not the same spectrum of disease

166) Pseudoxanthoma elasticum is caused by mutations in which of the following genes?

60

Page 15: ETAS_MCQ_03 b genodermatoses

A. Fibrillin 1

B. Collagen 5

C. ABCC6 gene Correct Choice

D. Lysyl oxidase

E. Elastin gene

Pseudoxanthoma elasticum is caused by mutations in the ABCC6 gene, which is an ATP-using cell transporter. Elastin and lysyl oxidase mutations cause cutis laxa, fibrillin 1 mutations cause Marfan syndrome, and collagen 5 mutations cause Ehlers-Danlos syndrome

167) A 4 year old boy presents with 2 soft, dark-blue, compressible nodules on her extremities. His mother has noted that these lesions have increased sweating and that they were present at birth. No one else in the family has had similar skin lesions. What step is indicated first to help determine the diagnosis?

A. Immediate referral to a gastroenterologist

B. Biopsy of a skin lesion

C. MRI of the abdomen

D. Stool guiacCorrect Choice

E. CBC

Blue rubber bleb nevus syndrome is described above. There are multiple venous malformations on the extremities and trunk, often present at birth to early childhood. The number of these lesions increase with age. The lesions may have increased sweating and can be combined with lymphatic-venous malformations. Skin lesions can be a clue to gastrointestinal venous malformations which can lead to secondary bleeding and anemia. The most reasonable screening test to determine if the patient has GI hemorrhage is a stool guiac. An MRI or complete blood count can be helpful, but are not the best test to start with. A skin biopsy is not indicated. If there is GI blood loss, evaluation by a gastroenterologist is useful

168) A patient presents with starfish keratoses, pseudoainhum, honeycombed PPK, and generalized ichthyosis. What is the most likely genetic defect?

A. Connexin 31

B. Connexin 26

C. LoricrinCorrect Choice

D. Connexin 30

61

Page 16: ETAS_MCQ_03 b genodermatoses

E. Connexin 33

The patient has Vohwinkel syndrome. This is an autosomal dominant syndrome with 2 clinical variants. The variant described above with generalized ichthyosis is due to a loricrin mutation. In the classic form with nonprogressive hearing loss connexin 26 is mutated

169) The best screening test for hemochromatosis is:

A. Complete Blood Count

B. FerritinCorrect Choice

C. Hematocrit

D. Copper levels

E. Transferrin

The ferritin is the best screening test for hemochromatosis, an autosomal disease of increased intestinal iron absorption. The other tests are not the best method for screening

170) A 32 year old woman is 5 weeks pregnant and is diagnosed with hyperthyroidism. Her doctor gives her a prescription for on methimazole 10 mg PO tid. Which of the following fetal abnormalities could be caused by this exposure?

A. Dermoid cyst

B. Aplasia cutis congenitaCorrect Choice

C. Spina bifida

D. Encephalocele

E. Meningocele

In-utero methimazole exposures has been linked to aplasia cutis congenita and should not be used in pregnant women. The FDA pregnancy class is D. The other listed options are not linked with maternal methimazole usage

171) Ichthyosiform erythroderma in lines of Blaschko, follicular atrophoderma, and stippled epiphyses are characteristic of which of the following disorders?

A. Refsum syndrome

B. Netherton syndrome

C. Erythrokeratodermia variabilis

62

Page 17: ETAS_MCQ_03 b genodermatoses

D. Sjogren-Larsson syndrome

E. Chondrodysplasia punctata Correct Choice

Chondrodysplasia punctata is a peroxisomal disorder caused by mutations in several genes that is associated with ichthyosiform erythroderma in lines of Blaschko, follicular atrophoderma, and stippled epiphyses

172) Yellow-brown depositions in Descemet's membrane of the corneas is diagnostic for:

A. Neurofibromatosis

B. Hemochromatosis

C. Hepatolenticular degenerationCorrect Choice

D. Pseudoxanthoma elasticum

E. Diabetes mellitus

Hepatolenticular degeneration or Wilson Disease is caused by a defect in biliary excretion of copper leading to accumulation in the liver, brain, cornea, pretibial hyperpigmentation, hepatomegaly and cirrhosis. The Kayser-Fleischer ring is the yellow-brown copper deposition in Descemet's membrane of the cornea. Other findings include ataxia, dysarthria and dementia. Hemochromatosis does not have an eye finding. Neurofibromatosis has Lisch nodules and Pseudoxanthoma elasticum has angioid streaks of the retina

173) The gene defect in Griscelli Syndrome is:

A. None of these answers are correct

B. Myosin Va or Rab27aCorrect Choice

C. LYST or CHS1

D. P gene

E. TRP1

Myosin Va or Rab27a are defective in Griscelli syndrome, an AR syndrome with mild albinism, pancytopenia, immunodeficiency, neurologic symptoms and an accelerated phase similar to Chediak-Higashi syndrome. LYST/CHS1 is defective in Chediak-Higashi syndrome. The P-gene is mutated in oculocutaneous albinism type 2 and TRP in oculocutaneous albinism type 3

174) Mutation in lamin A (nuclear envelope protein) has been found in:

A. Peutz-Jeghers syndrome

63

Page 18: ETAS_MCQ_03 b genodermatoses

B. Albright’s syndrome

C. Marfan syndrome

D. Buschek-Ollendorf syndrome

E. Progeria (Hutchinson-Gilford) Correct Choice

Progeria (Hutchinson-Gilford syndrome) is a sporadic condition characterized by lipoatrophy, sclerodermoid skin, alopecia, nail atrophy, craniomegaly with small face, muscle/bone wasting, and severe premature atherosclerosis resulting in early death. Recent studies have shown that mutations in nuclear envelope protein lamin A is associated with progeria

175) The characteristic dental findings in patients with tuberous sclerosis are:

A. Odontogenic cysts

B. Retention of primary teeth

C. Peg teeth

D. Anodontia

E. Enamel pitsCorrect Choice

Enamel pits are the characteristic dental findings in tuberous sclerosis. Peg teeth are found in incontinentia pigmenti and anhidrotic ectodermal dysplasia. Anodontia is found in hypomelanosis of ito and incontinentia pigmenti. Odontogenic cysts are seen in Gorlin syndrome, and retention of primary teeth is characteristic of Job syndrome

176) Urticaria pigmentosa is linked to a defect in the c-kit protooncogene. What autosomal dominant skin disease also has been linked to this defect?

A. PiebaldismCorrect Choice

B. Hypomelanosis of Ito

C. Incontinentia pigmenti

D. Waardenburg syndrome

E. Hermansky-Pudlak syndrome

Piebaldism is linked to a defect in the c-kit protooncogene. Hypomelanosis of Ito has whorled hypopigmentation, occasional CNS defects, scoliosis and anodontia, a sporadic mutation. Waardenburg syndrome has 4 types, 1-3 are AD, 4 is AR. Type 1 & 3 have defects in PAX-3, 2 in MITF, and 4 in Sox10, endothelin-3 ligand or receptor genes. Hermansky-Pudlak syndrome is AR and most commonly linked to defects in HPS, a lysosomal transport protein and AP3B1, a protein important in endocytic/exocytic sorting. Incontinentia pigmenti is an X-linked dominant syndrome with a defect in the NEMO gene

64

Page 19: ETAS_MCQ_03 b genodermatoses

177) Which of the following pairs of diseases are caused by mutations in DNA helicases?

A. Xeroderma pigmentosum and Cockayne syndrome

B. Muir-Torre syndrome and Birt-Hogg-Dube syndrome

C. Cowden syndrome and Bannayan-Riley-Ruvalcaba syndrome

D. Dyskeratosis congenita and Peutz-Jeghers syndrome

E. Bloom syndrome and Rothmund-Thompson syndrome Correct Choice

Bloom Syndrome and Rothmund-Thompson syndromes are caused by autosomal recessive mutations in DNA helicases. The common features include photodistributed poikiloderma. Patients with Bloom syndrome also demonstrate facial dysmorphism, hypogammaglobulinemia with recurrent respiratory and gastrointestinal tract infections, hypogonadism, leukemias and lymphomas, gastrointestinal adenocarcinomas, and oral/esophageal SCCs. Patients with Rothmund-Thompson syndrome develop premalignant acral keratoses, alopecia, nail dystrophy, cataracts, hypogonadism, and occasional solid tumors

178) Premature aging seen in Cockayne’s syndrome is caused by a mutation in which gene?

A. ATM gene

B. DNA helicase gene Correct Choice

C. NEMO gene

D. Patched gene

E. Transglutaminase

Cockayne’s syndrome is caused by a mutation in a DNA helicase gene, CSA or ERCC8. The condition is defined by growth deficiency, premature aging, and pigmentary retinal degeneration. 75% of patient have photosensitive eruptions and severe cataracts before the age of 3 are associated with poor prognosi

179) Which of the following metals is deficient in the serum of patients with Menkes kinky hair syndrome?

A. Biotin

B. Selenium

C. Zinc

D. Iron

65

Page 20: ETAS_MCQ_03 b genodermatoses

E. CopperCorrect Choice

Menkes kinky hair syndrome is transmitted in an X-linked recessive manner and is caused by a mutation in ATP7A, an ATP-dependent copper tranporter. This defect results in low serum levels of copper. These individuals will have hair abnormalities such as sparse, hypopigmented brittle hair, eyelashes and eyebrows, lax skin, a "cupid's bow" upper lip, CNS progressive deterioration, seizures, skeletal abnormalities and tortuous arteries. The other listed items are not associated with Menkes syndrome

180) Which of the following is NOT a complication of Kasabach-Merritt Syndrome:

A. Thrombocytopenia

B. Disseminated intravascular coagulation

C. Gastrointestinal bleeding

D. CHF

E. AtaxiaCorrect Choice

Kasabach-Merritt Syndrome results from platelet trapping due to the presence of a single or multiple large hemangiomas. Hematologic complications include thrombocytopenia, microangiopathic hemolytic anemia, DIC, and acute hemorrhage. The presence of large hemangiomas leads to high output failure (CHF) and can also compress on surrounding structures

181) Patients with progeria typically die of which of the following conditions?

A. Atherosclerotic heart disease Correct Choice

B. Metastatic carcinoma

C. Progressive systemic sclerosis

D. Infection

E. Nail atrophy

Progeria (Hutchinson-Gilford syndrome) is a sporadic condition characterized by lipoatrophy, sclerodermoid skin, alopecia, nail atrophy, craniomegaly with small face, muscle/bone wasting, and severe premature atherosclerosis resulting in early death

182) On cutaneous exam, angiokeratoma corporis diffusum is characteristic of which of the following conditions?

A. Fabry disease

B. Sialodosis

66

Page 21: ETAS_MCQ_03 b genodermatoses

C. Fucosidosis

D. All of these options are correctCorrect Choice

E. None of these options are correct

Findings of angiokeratoma corporis diffusum are found in all three listed conditions. They cannot by distinguished by skin exam

183) The histologic image shown in this picture is associated with which of the following syndromes?

A. Dyskeratosis congenita

B. Gorlin syndrome

C. Banayan-Riley-Ruvalcaba syndrome

D. Chediak-Higashi syndrome

E. Multiple hamartoma syndromeCorrect Choice

The histologic picture shown is that of a sclerotic fibroma with the characteristic storiform/whorled pattern of collagen. The lesion has low cellularity with spindle cells. Banayan-Riley-Ruvalcaba syndrome shares the same gene defect as Multiple hamartoma syndrome (Cowden syndrome) but is not associated with sclerotic fibromas. Dyskeratosis congenita is not associated with sclerotic fibromas

184) A patient with Crowe’s sign and an optic glioma has which of the following disorders?

A. Tuberous sclerosis

B. Neurofibromatosis II

C. Watson syndrome

D. Lester iris syndrome

E. Neurofibromatosis I Correct Choice

The diagnostic criteria for neurofibromatosis I include meeting 2 or more of the following 7 criteria: (1) >5 café au lait macules (CALMs) that are >5mm in a prepubertal person or >15 mm in a postpubertal person, (2) >1 neurofibroma or 1 plexiform neurofibroma, (3) axillary/inguinal freckling (Crowe’s sign), (4) optic glioma, (5) >1 Lisch nodule (iris hamartoma), (6) sphenoid dysplasia, (7) 1st degree relative with neurofibromatosis I

185) The most common cutaneous neoplasm associated with Muir Torre Syndrome is:

67

Page 22: ETAS_MCQ_03 b genodermatoses

A. Basal cell carcinoma with sebaceous differentiation

B. Sebaceous adenomaCorrect Choice

C. Keratoacanthoma

D. Sebaceous carcinoma

E. Squamous cell carcinomas

Muir-Torre syndrome is an autosomal dominant disorder due to a DNA mismatch repair gene (MSH2). Patients present with sebaceous tumors, with adenomas being the most common (sebaceous carcinomas next common) and associated visceral malignancies such as colon cancer (most common).

186) The finding of 'maltese crosses' in the urine is characteristic of which of the following conditions?

A. Fabry diseaseCorrect Choice

B. Alkaptonuria

C. Neimann-Pick disease

D. Hunter syndrome

E. Gaucher disease

The 'maltese cross' finding in urine is characteristic of Fabry disease. Alkaptonuria will show dark urine with a pH > 7.0. There are no urinary findings in Hunter syndrome, Gaucher or Neimann-Pick disease

187) The gene PTEN (phosphatase and tensin homolog deleted on chromosome ten) is implicated in which of the following syndromes?

A. Cowden

B. None of the options are correct

C. Banayan-Riley-Ruvalcaba

D. All of the options are correctCorrect Choice

E. Proteus

Mutations in the PTEN gene are implicated in all three syndromes listed. There are conflicting reports in the literature regarding the linkage of Proteus and PTEN mutations. For further details, see OMIM #176920

68

Page 23: ETAS_MCQ_03 b genodermatoses

188) Which cutaneous finding is seen in patients with phenylketonuria?

A. Pigment dilution of hair and skinCorrect Choice

B. Ichthyosis

C. Phyrnoderma

D. Angular stomatitis

E. Erosive diaper dermatitis

Phenylketonuria is an autosomal recessive condition caused by a mutation in the gene coding for phenylalanine hydroxylase. Defect in this enzyme results in accumulation of phenylalanine and its metabolites. Increased phenylalanine has toxic effects on the central nervous system in addition to competitively inhibiting tyrosine in melanogenesis. Inhibition of melanogenesis results in pigmentary dilution of the hair and skin. Other features of this condition include a predisposition to eczema, sclerodermoid changes of the skin, urine that has a distinctive "mousy" odor, psychomotor delay, mental retardation, seizures and hyperreflexia. A low-phenylalanine diet instituted early on can prevent these manifestations of the disease. The morbidity of phenylketonuria has improved since the advent of routine neonatal screening for this condition

189) All of the following disorders are exacerbated by UV radiation except:

A. Rothmund-Thomopson syndrome

B. Hartnup’s disease

C. Bloom syndrome

D. Cockayne syndrome

E. Refsum syndromeCorrect Choice

Refsum’s syndrome is an autosomal recessive disorder caused by mutations in phytanoyl-CoA hydroxylase. Clinically, patients have mild icthyosis, cerebellar ataxia, polyneuropathy, salt and pepper retinitis pigmentosa, sensorineural deafness, and arrhythmias with heart block. They are not overly sensitive to UV radiation

190) Angioid streaks on retinal exam are characteristic of which of the following syndromes?

A. Pseudoxanthoma elasticumCorrect Choice

B. Choroid malformations

C. Salt & pepper retinitis pigmentosa

D. Eyelid papillomas

69

Page 24: ETAS_MCQ_03 b genodermatoses

E. Lester iris

Angioid streaks are characteristic of pseudoxanthoma elasticum. They are caused by rupture of Bruch's membrane of the choroid. Choroid malformations are found in Sturge-Weber syndrome, eyelid papillomas in xeroderma pigmentosum, Lester iris in Nail-patella syndrome and salt & pepper retinitis pigmentosa in Refsum syndrome

191) Which one of the following is the most common oncogenic virus in patients with epidermodysplasia verruciformis?

A. HPV-16

B. HPV-5 Correct Choice

C. HPV-13

D. HPV-8

E. HPV-33

Epidermodysplasia verruciformis is a rare autosomal recessive disorder in which an impaired cellular immunity allows widespread infection with certain subtypes of the human papilloma virus (HPV). Some of these lesions have a tendency for malignant transformation, most commonly those verruca caused by HPV type 5

192) Crumpled ears are associated with which disorder?

A. Marfan syndrome

B. Buschke-Ollendorf syndrome

C. Cutis laxa

D. Ehlers-Danlos Syndrome

E. Congenital contractural arachnodactyly Correct Choice

Congenital contractural arachnodactyly is an autosomal dominant disorder caused by mutations in fibrillin 2. Affected patients have long limbs, arachnodactyly, scoliosis, and crumpled ears

193) Which of the following conditions is inherited in an X-linked dominant (XD) manner?

A. Focal dermal hypoplasia

B. Chodrodysplasia punctata

C. None of the answers are correct

70

Page 25: ETAS_MCQ_03 b genodermatoses

D. All of the answers are correctCorrect Choice

E. CHILD Syndrome

All of the syndromes listed are XD. Other XD syndromes are: Incontinentia pigmenti and Bazex syndrome

194) What is the characteristic radiographic finding in type I Gaucher disease?

A. Osteopoikilosis

B. Melorheostosis

C. Enchondromas

D. Ehrlenmeyer flask deformityCorrect Choice

E. Supernumerary vertebrae with extra ribs

The Ehrlenmeyer flask deformity is found in the femoral midshaft as well as aseptic necrosis of the femoral head and widening of the distal femur. Endochondromas are seen in Maffucci syndrome, Osteopoikilosis in Buschke-Ollendorf syndrome, Melorheostosis (linear hyperostosis under affected skin) in linear scleroderma and supernumerary vertebrae with extra ribs in incontinentia pigmenti

195) A child presents with pretibial hyperpigmentation, ataxia, decreased motor coordination, cirrhosis, and decreased motor coordination. The physical exam which would reveal the most specific finding for this disease is:

A. Slit-lamp eye examCorrect Choice

B. Renal ultrasound

C. Hearing test

D. Colonoscopy

E. EKG

Wilson’s disease (also known as hepatolenticular degeneration) is an autosomal recessive disorder result in defective biliary excretion of copper, leading to copper accumulation in the liver, brain and cornea. Clinical features include hepatomegaly, cirrhosis, ataxia, dysarthria, decreased motor coordination, pretibial hyperpigmentation, blue lunulae, and copper deposition in the cornea—Kayser-Fleisher ring, which can be diagnosed using a slit-lamp

196) Eyelid string of pearls are seen in which of the following conditions?

A. Hutchinson-Gilford syndrome

71

Page 26: ETAS_MCQ_03 b genodermatoses

B. Lipoid proteinosis Correct Choice

C. Focal dermal hypoplasia

D. Gaucher’s disease

E. Beare-Stevenson cutis gyrata syndrome

Lipoid proteinosis is an autosomal recessive condition characterized by yellow papules on the face and oropharynx, eyelid string of pearls, hoarse voice, verrucous nodules of elbows and knees, and bean-shaped temporal and hippocampal calcification with occasional seizures. Histologically, there are PAS+ deposits in the affected tissue

197) What finding is seen on brain imaging of patients with Papillon-Lefevre Syndrome?

A. calcification of the falx cerebri

B. calcification of the hippocampus

C. tram track calcifications

D. calcification of the duraCorrect Choice

E. agenesis of the corpus callosum

Pappilon Lefevre Syndrome is an autosomal recessive syndrome characterized by transgredient PPK and periodontitis. There is a defect in cathepsin C. One sees dural calcification at the tentorium and choroid attachments. Tram track calcifications are seen in STurge-WEber. CAlcification of the falx cerebri and agenesis of the corpus callosum is seen in basal cell nevus syndrome. Hippocampal calcification is seen in lipoid proteinosi

198) A patient with Klinefelter Syndrome may be expected to experience which of the following:

A. Recurrent leg ulcersCorrect Choice

B. Scarring alopecia

C. Recurrent pulmonary infections

D. Gastroesophageal reflux

E. Pulmonary valve stenosis

Klinefelter syndrome results from nondisjunction during meiosis, leading to the XXY genotype. Patients are characteristically tall (long lower extremities) with scant body and pubic hair. Klinefelter patients have numerous varicosities predisposing them to recurrent leg ulcers

199) Which of the following is the first symptom of ataxia telangiectasias?

72

Page 27: ETAS_MCQ_03 b genodermatoses

A. Facial telangiectases

B. Conjunctival telangictases

C. Breast cancer

D. Hematologic malgignancy

E. Cerebellar ataxia Correct Choice

Ataxia Telangiectasia (Louis-Bar syndrome) is an autosomal recessive disorder usually caused by mutations in the ATM gene, which is a chromosomal strand break repair enzyme. Cerebellar ataxia is the first sign, followed by telangiectases of the conjunctiva and skin. Thymic hypoplasia predisposes to increased infections. There is increased sensitivity to ionizing radiation resulting in hematologic and solid tumors. Female carriers have increased risk of breast cancer

200) Which type of epidermolysis bullosa simplex is associated with early death?

A. Weber-Cockayne

B. Dowling-Maera Correct Choice

C. Non-Herlitz variant

D. Ogna variant

E. Generalized (Koebner)

The Dowling-Maera variant of epidermolysis bullosa simplex is associated with widespread bullae, significant mucous membrane and laryngeal/esophageal involvement, nail dystrophy, and early death

201) Patients with Darier’s disease are at increased risk for:

A. Melanoma

B. Basal cell carcinoma

C. Lipid abnormalities

D. Kaposi’s varicelliform eruptionCorrect Choice

E. Decreased life span

Kaposi’s varicelliform eruption is the condition in which viral infection occurs in a patient with a pre-existing chronic dermatitis. Darier’s disease is an autosomal dominant genodermatosis caused by a mutation in ATP2A2 which encodes SERCA2. Cutaneous manifestations of warty, hyperkeratotic papules in a seborrheic dermatitis, which may be infected with HSV or bacteria

73

Page 28: ETAS_MCQ_03 b genodermatoses

202) A patient with port wine stain on a lower extremity, hemihypertrophy of the limb and lymphatic and deep venouse insufficiency of the affected limb would be considered to have Klippel-Trenaunay-Weber syndrome. What additional feature would need to be present to define the patient as having Parkes-Weber syndrome?

A. Macroglossia

B. Multiple cafe-au-lait macules

C. Arteriovenous fistulasCorrect Choice

D. Cutis marmorata

E. Distichiasis

Parkes-Weber syndrome has the additional feature of arteriovenous fistulas. The remaining features are not part of these syndromes

203) What is the first sign seen in children with ataxia telangectasia syndrome?

A. nystagmus

B. cutaneous and bulbar conjunctival telangectasias Correct Choice

C. recurrent viral or bacterial infections

D. oculomotor apraxia

E. cerebellar ataxia

Telangectasias are the first sign of ataxia telangectasia syndrome (Louis-Bar). Other findings other than those listed above include: increased risk of lymphoma or breast carcinoma in heterozygotes, granulomas, cafe au lait macules. IgA, IgG2 and IgE will be decreased or absent. There is an increased sensitivity to ionizing radiation

204) A patient with this autosomal recessive disorder caused by a defect in helicase is an increased risk for which malignancy?

A. Medullary thyroid carcinoma

B. Renal cell carcinoma

C. Acute leukemiaCorrect Choice

D. Squamous cell carcinoma of the lung

E. Prostate carcinoma

74

Page 29: ETAS_MCQ_03 b genodermatoses

Bloom's syndrome is an autosomal recessive disorder caused by a mutation in DNA helicase. It is characterized by photodistributed erythema in a butterfly distribution, malar hypoplasia with a prominent nose, high pitched voice, and an increased risk for malignancy (acute leukemia, lymphoma, and GI adenocarcinoma

205) In biopsies from blisters in patients with junctional epidermolysis bullosa, the split is found in the:

A. Basal cell layer of the epidermis

B. Lamina densa

C. Lamina lucidaCorrect Choice

D. Squamous cell layer of the epidermis

E. None of the answers are correct

The split seen in junctional epidermolysis bullosa is in the lamina lucida. The other locations can be involved in blistering disease, but not junctional epidermolysis bullosa

206) The NEMO gene is defective in Bloch-Sulzberger syndrome. What other syndrome has been linked with defects in the NEMO gene?

A. Hypomelanosis of Ito

B. Tuberous sclerosis

C. Piebaldism

D. Waardenburg syndrome

E. Hypohidrotic ectodermal dysplasia with immune deficiency Correct Choice

Hypohidrotic ectodermal dysplasia with immune deficiency, is caused by mutations in the NEMO (IKK-gamma gene). As opposed to the X-linked dominant inheritance of Bloch-Sulzberger syndrome (incontinentia pigmenti), this is a X-linked recessive disorder. Hypomelanosis of Ito is sporadically inherited and is not linked with a gene defect. Tuberous sclerosis is autosomal dominant and has been linked to defects in tuberin and hamartin tumor suppressor genes. Waardenburg syndrome has four subtypes, linked with the PAX-3, MITF and SOX10/endothelin-3 receptor genes. Piebaldism is linked to defects in the c-kit protooncogene

207) What is the gene defect in harlequin fetus?

A. transglutaminase

B. none of these answers are correct

75

Page 30: ETAS_MCQ_03 b genodermatoses

C. ABCC6

D. steroid sulfatse

E. ABCA12Correct Choice

Harlequin fetus is an autosomal recessive disorder. The gene defect is ABCA12

208) Regarding the inheritance of Ehlers-Danlos syndrome, which subtype is inherited in an X-linked recessive manner?

A. The type with the lysyl oxidase deficiency Correct Choice

B. The type with a collagen 5 defect

C. The type with the lysyl hydroxylase deficiency

D. The type with a defect in procollagen aminopeptidase

E. The type with a collagen 3 defect

Lysyl oxidase deficiency is related to type V or X-linked Ehlers-Danlos syndrome (EDS). Type IX, also has x-linked recessive inheritance with mild symptoms of EDS with occipital exostoses and hernias. Lysyl hydroxylase deficiency is seen in type VI EDS, linked with severe kyphoscoliosis, retinal detachment and other eye abnormalities. Collagen V deficiencies are seen in type I (Gravis) and type II (Mitis) EDS, associated with skin elasticity, gaping wounds, hypermobile joints, Gorlin’s sign, blue sclera and mitral valve prolapse

209) Anodontia is a bone finding seen in which of the following conditions:

A. Tuberous sclerosis

B. Letterer-Siwe disease

C. Hyper-IgE syndrome

D. Jackson Sertoli syndrome

E. Hypomelanosis of ItoCorrect Choice

Hypomelanosis of Ito, or Incontinentia pigmenti achromians is a condition characterized by marble-cake hypopigmentation, epilepsy, alopecia, scoliosis and mental/motor retardation. The characteristic dental abnormality is anodontia. The remaining syndromes are not associated with anodontia

210) Multiple cylindromas and trichoepitheliomas are associated with which of the following syndromes?

76

Page 31: ETAS_MCQ_03 b genodermatoses

A. Gardner

B. Cronkhite-Canada

C. Brook-SpieglerCorrect Choice

D. Cowdens

E. Nicolau-Balus

These findings are characteristic of Brook-Spiegler syndrome. Gardner syndrome is associated with gastrointestinal carcinoma and adnexal neoplasms, Cronkhite-Canada syndrome, Nicolau-Balus and Cowdens are not associated with cylindromas or trichoepitheliomas

211) Which of the following features is not associated with Cornelia de Lange Syndrome?

A. Normal intelligenceCorrect Choice

B. Fifth finger clinodactyly

C. Recurrent lung infections

D. Cryptorchidism

E. Characteristic facies with downturned mouth, hirsutism, synophrys, trichomegaly, anteverted nostrils, long philtrum and low set ears

Children with Cornelia de Lange are usually severly retarded with an IQ <35. In addition to the features listed above, other features include cutis marmorata, hypoplastic nipples and umbilicus, low-pitched cry in infancy and congenital heart defects. While most cases are inherited in a sporadic manner, those cases which are familial are thought to be autosomal dominant and associated with the NIPBL (nipped-beta-like) gene. Prognosis is poor with premature death often secondary to sspiration or recurrent pulmonary infection

212) Erythematous keratotic plaques of KID Syndrome most commonly occur in which location?

A. Chest

B. Neck

C. Back

D. FaceCorrect Choice

E. Abdomen

The erythematous keratotic plaques of KID Syndome occur on face, extremities > trunk. KID Syndrome is characterized by keratitis, icthyosis, and deafness. It is inherited in an autosomal dominant fashion and caused by a mutation in the GJB2 gene which encodes connexin 26

77

Page 32: ETAS_MCQ_03 b genodermatoses

213) Findings of milia, cylindromas and the condition shown in the pathology image are characteristic of which of the following syndromes?

A. Brook-Spiegler syndrome

B. Gorlin's syndrome

C. Rombo syndrome

D. Rasmusen syndromeCorrect Choice

E. Familial cylindromatosis

Rasmusen syndrome is characterized by milia, trichoepitheliomas and cylindromas. The other options have trichoepitheliomas as a feature of the syndrome, but not with the other listed findings

214) In addition to pheochromocytoma and medullary thyroid carcinoma and the skin finding in the image in a patient with a marfanoid body habitus is associated with which of the following syndromes?

A. MEN type I

B. Basex syndrome

C. Cowden syndrome

D. MEN type IIaCorrect Choice

E. MEN type IIb

The image show amyloidosis, which is found in addition to pheochromocytoma, medullary thyroid carcinoma and parathyroid abnormalities in Sipple syndrome or MEN type IIa. Cowden disease is not associated with any of the listed findings.

215) Argininosuccinic aciduria is associated with a hair abnormality shown in the image. Which of the following hair finding is seen in these patients?

A. Trichorrhexis nodosaCorrect Choice

B. Trichoschisis

C. Trichorrhexis invaginata

D. Pili torti

E. None of these options are correct

78

Page 33: ETAS_MCQ_03 b genodermatoses

Trichorrhexis nodosa is shown in the figure and is found in ~50% of affected patients with argininosuccinic aciduria. Affected individuals will have short, broken scalp hairs, often increased with late onset disease. Usually these findings are increased in the occipital region of the scalp. The remaining options are not found in affected individual

216) The most common cutaneous association with monilethrix is:

A. Atrophy

B. Eczema

C. Hypopigmentation

D. Keratosis PilarisCorrect Choice

E. Hyperpigmentation

Monilethrix is an autosomal dominant condition which, by definition, presents with “beaded” hear. Clinically, patients present with short, sparse lusterless hair. Keratosis pilaris is the most common associated feature

217) An infant with doughy, redundant skin and short sparse hairs is likely to show which features on x-ray?

A. Osteopoikilosis

B. Sphenoid wing dysplasia

C. Stippled epiphyses

D. Periosteal thickening

E. Metaphyseal widening in the long bonesCorrect Choice

The patient described has Menkes Kinky Hair syndrome, an X-linked recessive disease due to a defect in an intestinal copper transport protein. Clinical features include pili torti, short, brittle “steel-wool” hair, and spare eyelashes and sparse broken eyebrows. The skin is often hypopigmented with a soft, “doughy” consistency and redundancy. Musculoskeletal manifestations include metaphyseal widening with spurs in the long bones

218) Familial macular and lichen amyloidosis is a feature of which of the following conditions?

A. Sipple syndrome Correct Choice

B. Peutz-Jeghers syndrome

C. Birt-Hogg-Dube syndrome

79

Page 34: ETAS_MCQ_03 b genodermatoses

D. Dyskeratosis congenita

E. Marfan syndrome

Sipple syndrome (MEN 2a) is caused by autosomal dominant mutations in the ret protooncogene. Patients develop parathyroid cancers, pheochromocytomas, and medullary cancer of the thyroid gland. Familial macular and lichen amyloidosis is also a feature of this syndrome

219) Which of the following is a feature of Neurofibromatosis type II?

A. Lisch nodules

B. Optic gliomas

C. Juvenile posterior subcapsular lenticular opacities Correct Choice

D. Congenital hypertrophy of the retinal pigment epithelium

E. Lester iris

Neurofibromatosis type II is an autosomal dominant disorder caused by mutations in schwannomin/merlin. Clinical features include cutaneous schwannomas and neurofibromas, bilateral vestibular schwannomas, and juvenile posterior subcapsular lenticular opacities

220) Findings of dysplastic nevi and melanoma inherited in an autosomal dominant fashion is linked with which of the following?

A. Pancreatic malignancyCorrect Choice

B. Renal cell carcinoma

C. Breast malignancy

D. Thyroid malignancy

E. Colon carcinoma

Familial dysplastic nevi/melanoma syndrome is linked with increased risk of pancreatic cancers and astrocytomas. There are no reported increased risks for the other types of cancers listed

221) Which gene is defective in Wiskott-Aldrich syndrome?

A. NCF2

B. NCF1

C. WASCorrect Choice

80

Page 35: ETAS_MCQ_03 b genodermatoses

D. CYBB

E. CYBA

The WAS gene is defective in Wiskott-Aldrich syndrome. WAS is an Arp2/3 complex interacting protein. The remaining options are genes related to Chronic Granulomatous Disease and are not active in the pathogenesis of Wiskott-Aldrich syndrome

222) Refsum syndrome is due to a deficiency in phytanyl coenzyme A hydroxylase. Treatment for this condition is:

A. Diet low in green vegetables, dairy and ruminant fats Correct Choice

B. No treatment is available at this time

C. Enzyme replacement

D. Avoid phenylalanine

E. Diet high in green vegetables, dairy and ruminant fats

Treatment is with a diet low in green vegetables, dairy and ruminant fats is the treatment of choice for Refsum syndrome. Avoidance of specific amino acids is not helpful

223) Dermatofibrosis lenticularis disseminata and osteopoikilosis are findings seen with mutations of which of the following genes?

A. Lysyl hydroxylase

B. Lysyl oxidase

C. LEMD3Correct Choice

D. Fibrillin 2

E. ABCC6

Buschke-Ollendorf syndrome is caused by a loss-of-function mutation in LEMD3

224) An infant girl of short stature and shortened 4th and 5th metacarpals is being evaluated for coarctation of the aorta and horseshoe kidneys. Physical examination most likely reveals:

A. Arachnodactyly

B. Webbed neckCorrect Choice

C. Hemangioma

81

Page 36: ETAS_MCQ_03 b genodermatoses

D. Giant congenital melanocytic nevus

E. Alopecia

Turner’s syndrome results from nondysjunction during gametogeneiss leading to the XO genotype. Clinical features include short stature, redundant neck folds/webbed neck, multiple pigmented nevi, low set hairline, triangular facies, low-set ears, ptosis, wide-set nipples, shortened 4th and 5th metacarpals, hypoplasia of lymphatics, coarctation of the aorta, and horseshoe kidneys

225) Mutations in nuclear lamins are associated with which clinical feature?

A. Lipomatosis

B. Sarcomas

C. Aplasia cutis congenita

D. Lipodystrophy Correct Choice

E. Lymphedema

Bernardinelli-Seip congenital lipodystrophy and familial partial lipodystrophy are caused by mutations in nuclear lamins. The former is characterized by generalized lipodystrophy and the latter is characterized by by partial lipodystrophy

226) A patient presents with multiple tumors on the scalp with pathology shown. Which of the following genes is most likely mutated?

A. NEMO

B. APC

C. CYLDCorrect Choice

D. SPINK5

E. PTEN

The pathology shown is that of a cylindroma. Multiple cylindromas of the scalp, associated with eccrine spiradenomas can occur in the autosomal dominant familial cylindromatosis sydrome. The remaining genes are not associated with formation of cylindromas

227) The most common ocular association with cutis marmorata telangiectatica congenital is:

A. Cataracts

B. Retinoblastoma

82

Page 37: ETAS_MCQ_03 b genodermatoses

C. GlaucomaCorrect Choice

D. Corneal opacity

E. Angioid streaks

Glaucoma is the most common associated eye finding in CMTC patients. Glaucoma is also seen in patients with neurofibromatosis type 1 and Sturge Weber patients

228) The following enzyme defect is most commonly seen in CHILD Syndrome.

A. 3-beta-hydroxysteroid isomerase

B. NAD oxido reductase

C. 3-beta-hydroxysteroid dehydrogenaseCorrect Choice

D. Aryl sulfatase E

E. DNA helicase

CHILD Syndrome is a X-linked dominant disorder characterized by unilateral ichthyosiform erythroderma, ipsilateral limb deformity, and ipsilateral organ hypoplasia. The most commom gene defect is NSDHL which encodes 3-beta hydroxysteroid dehydrogenase. EBP gene defects which encode 3-beta-hydroxysteroid isomerase have been described, however this is the usual defect in Conradi-Hunermann Syndrome. Aryl sulfatase E is mutated in X-linked recessive chondrodysplasia punctata

229) Which of the following syndromes demonstrate atrophoderma vermiculatum?

A. All of these answers are correctCorrect Choice

B. Nicolau-Balus Syndrome

C. ROMBO Syndrome

D. Tuzun Syndrome

E. Braun-Falco-Marghescu Syndrome

All of the listed syndromes include atrophoderma vermiculatum as part of their constellation of symptoms. Atrophoderma vermiculatum is characterized by honeycomb pattern of atrophic scars on the face. Tuzun Syndrome also has scrotal tongue. ROMBO has BCCs, milia, peripheral vasodilation, trichoepitheliomas. Nicolau-Balus has eruptive syringomas and milia. Braun-Falco-Marghescu has keratosis pilaris and palmoplantar hyperkeratosis

230) Which opthamologic disease is associated with this disorder?

83

Page 38: ETAS_MCQ_03 b genodermatoses

A. Posterior subcasular lentiular opacity

B. Cataracts

C. GlaucomaCorrect Choice

D. Ectopia lentis

E. Retinitis pigmentosa

Sturge-Weber syndrome is a sporadic disease characterized by a capillary malformation in the trigeminal distribution. Patients may have associated cerebral atrophy, vascular malformations of the leptomeninges, and seizures. All patients with Sturge-Weber should be referred to the opthamologist for glaucoma screening

231) Ivory-colored papules between the angles of the scapulae are characteristic of which syndrome:

A. Scheie

B. Morquio

C. HunterCorrect Choice

D. Hurler

E. Sanfilippo

These syndromes are all mucopolysaccharidoses. These papules are characteristic of Hunter syndrome which is caused by a deficiency in iduronate sulfatase

232) You receive a hospital consult from the gastroenterology service for a 42-year old woman with esophageal cancer. They would like your opinion on the yellow, thickened areas on her palms and soles in areas of pressure. When you speak with her, she says that her father had similar problems and it runs in her family. Which of the following is defective?

A. Connexin 31

B. Connexin 30.3

C. TOC geneCorrect Choice

D. Plakoglobin

E. Desmoplakin

This case describes Howell-Evans syndrome. This AD syndrome characteristically has a PPK in areas of pressure, oral leukoplakia and esophageal carcinomas. Desmoplakin is defective in Carvajal syndrome and plakoglobin in Naxos syndrome. Connexin 31 and 30.3 are linked to erythrokeratoderma variabilis, which includes a PPK, but not esophageal carcinoma

84

Page 39: ETAS_MCQ_03 b genodermatoses

233) Defects in Fibrillin 2 are linked with:

A. Congenital contractural arachnodactylyCorrect Choice

B. Lipoid proteinosis

C. Arthrochalasis multiplex congenita

D. Occipital horn syndrome

E. Cutis Laxa

Fibrillin 2 defects arelinked primarily with congenital contractural arachnodactyly. This syndrome is associated with long limbs, arachnodactyly, scoliosis and crumpled ears. Occasionally, fibrillin 2 can be associated with Marfan syndrome also. The other conditions are not linked to fibrillin mutations

234) A patient with multiple deeply pigmented papules has a skin biopsy which reveals an epitheloid blue nevus. The next appropriate step is:

A. Order an echocardiogramCorrect Choice

B. Refer to genetics

C. Schedule prophylactic excision of the lesion

D. Begin a malignancy work-up

E. Reassure the patient and follow up as needed

Epithelioid blue nevi have been reported with and without association with cardiac myxomas as a component of the Carney complex (NAME/LAMB syndrome). Carney complex is an autosomal dominant disorder caused by mutations in PRKAR1A. Patients have cutaneous and atrial myxomas, blue nevi, ephelides, adrenocortical disease, and testicular tumors

235) The nucleotide excision DNA repair pathway is defective in which disease

A. Xeroderma pigmentosaCorrect Choice

B. Bourneville's disease

C. Severe combined deficiency syndrome

D. Griscelli syndrome

E. Sjogren-Larssen syndrome

The pathogensis of Xeroderma Pigmentosum shows mutations i genes encoding DNA repair enzymes, leading to defective DNA excision repair upon exposure to UV radiation (D). Severe

85

Page 40: ETAS_MCQ_03 b genodermatoses

combined deficiency syndrome-major defect in cell-mediated and humoral immunity; most lack antibody-dependentcellular cytotoxicity and natural killer cell function (thus (B) is incorrect). The pathogensis for Griscelli Syndrome is a mutation in gene encoding for myosin Va or RAB27 a (thus (C) is incorrect). Sjogren-Larsson Syndrome has mutations in the FALDH gene (thus (E) is incorrect). Bourneville's syndrome (AKA Tuberous Sclerosis) shows a mutation in either TSC1 ancoding hamartin or TSC2 encoding tuberin (thus (A) is incorrect

236) A child presenting with the scalp findings shown and a right arm hypoplasia would be diagnosed with which of the following syndromes?

A. Bart's syndrome

B. Progeria

C. Adams-Oliver syndromeCorrect Choice

D. None of these options are correct

E. Dunnigan syndrome

Adams-Oliver syndrome is defined by aplasia cutis congenita (ACC) (shown in the image), usually of the midline scalp with limb hypoplasia. Bart's syndrome also has ACC as a finding, but it is usually present on the lower extremities and associated dominant dystrophic epidermolysis bullosa. Progeria is a premature aging syndrome associated with a lamin-A mutation. Dunnigan syndrome is also known as Familial partial lipodystrophy and is associated with a mutation in the BSCL2 gene. Neither are associated with findings of ACC

237) Papillon-Lefevre and Haim-Munk syndromes have which of the following symptoms?

A. Pseudoainhum

B. Esophageal cancer

C. Eccrine syringofibradenoma

D. Right-ventricular cardiomyopathy

E. Periodontitis with tooth loss Correct Choice

Right-ventricular cardiomyopathy is associated with Naxos syndrome, pseudoainhum is associated with Vohwinkel syndrome, esophageal cancer is associated with Howel-Evans syndrome, and eccrine syringofibradenomas are associated with Schopf-Schulz-Passarge syndrome. Periodontitis with tooth loss is associated with Papillon-Lefevre and Haim-Munk syndromes, which are caused by mutations in Cathepsin C

238) The development of which malignancy is most commonly associated with lymphomatoid papulosis?

A. non-Hodgkin's lymphoma

86

Page 41: ETAS_MCQ_03 b genodermatoses

B. immunoblastic lymphoma

C. mycosis fungoidesCorrect Choice

D. Waldenstrom's macroglobulinemia

E. multiple myeloma

Lymphomatoid papulosis is a recurrant eruption of unclear etiology characterized by the appearance of red-brown papules and nodules which spontaneously disappear in 3 to 8 weeks. It is notable for histologic features which suggest a CD30 positive malignant lymphoma. There is controversy regarding whether lymphomatoid papulosis (LyP) is a malignant, premalignant or benign condition. The disease may last from months to years and in up to 20% of patients, it may be preceded by, followed by, or associated with another type of cutaneous malignancy. Generally, this is mycosis fungoides, a CD30-positive large T-cell lymphoma or Hodgkin's disease. Because of this potential risk, long-term follow-up of these patients is required.

(Fitzpatrick's Dermatology in General Medicine, 5th ed 1999 p1269)(Bolognia Dermatology 1st ed 2003, p1935-1937)

239) Pseudoxanthoma elasticum (PXE) can be transmitted in an autosomal dominant, recessive or sporadic manner. Which of the following genes is mutated in PXE?

A. Collagen III

B. Collagen I

C. ABCC6Correct Choice

D. MAN1

E. Collagen V

ABCC6 is mutated in PXE. Other findings include fragmented and calcified elastin in skin, eyes, arteries. There is the appearance of plucked chicken skin on the flexures and yellow papules on the mucous membranes. Angioid streaks are present in the eye. Other findings include gastric hemorrhage and arterial disease. The others are not involved in PXE

240) Pili trianguli et canaliculi is characteristic of which of the following syndromes?

A. Uncombable hair syndromeCorrect Choice

B. Bjornstad syndrome

C. Menkes kinky hair syndrome

D. Netherton's syndrome

E. Leiners disease

87

Page 42: ETAS_MCQ_03 b genodermatoses

Uncombable hair syndrome is characterized by pili trianguli et canaliculi. On examination of the hair, it is triangular with a canal-like groove runs along the shaft. The clinical findings are that of blond, shiny, "spun glass" hair. It is an autosomal dominant syndrome with no known gene locus. The remaining syndromes do not have this hair finding present

241) A BSCL2 gene mutation with the cutaneous findings of generalized lipodystrophy, hyperlipemia, hepatomegaly, acanthosis nigricans, elevated basal metabolic rate and non-ketotic insulin resistant diabetes mellitus are characteristic of which of the following syndromes?

A. None of the answers are correct

B. Familial partial lipodystrophy

C. All of the answers are correct

D. Bjornstad syndrome

E. Berardinelli-Seip congenital lipodystrophyCorrect Choice

Berardinelli-Seip congenital lipodystrophy is described above. Familial partial lipodystrophy is characterized by a defect in LMNA and has symmetric lipoatrophy of trunk and limbs with sparing of neck, shoulders, buffalo hump area and genitalia, tuboeruptive xanthomas, acanthosis nigricans and hypertriglyceridemia. Bjornstad syndrome is characterized by pili torti and deafness

242) Collagen III is mutated in which type(s) of Ehlers-Danlos syndrome (EDS)?

A. Periodontitis (type VIII)

B. None of the answers are correct

C. Vascular (type IV)

D. Benign Hypermobile (type III)

E. All of the answers are correctCorrect Choice

Collagen III is mutated in all three types of EDS listed. Benign hypermobile type EDS is associated with hypermobile joints and is autosomal dominant (AD) in transmission. Vascular type EDS is associated with arterial and visceral rupture leading to early death, and visible venous patterns. It is transmitted autosomal recessive (AR) or AD. Periodontitis type EDS is associated with mild EDS symptoms and periodontitis

243) What is the classic radiologic findings associated with this disorer?

A. Calcifications of the falx-cerebri

B. Osteopatha striata

88

Page 43: ETAS_MCQ_03 b genodermatoses

C. Dural calcifications

D. Osteopoikilosis

E. Tram-track calcifications of the temporal and occipital cortexCorrect Choice

Sturge-Weber syndrome is a sporadic disroder characterized by a facial capillary malformation in a trigeminal nerve distribution. Patients with Sturge-Weber may have cerebral atrophy, ipsilateral vascular malformations of the leptomeninges, seizures, and glaucoma. The classic radiologic finding is tram-track calcifications of the temporal and occipital cortex

244) Findings of eyelid papules (string of pearls) and a hoarse cry in infants is characteristic of which of the following syndromes?

A. Amyloidosis

B. Pseudoxanthoma elasticum

C. None of these answers are correct

D. Lipoid proteinosisCorrect Choice

E. Disseminated xanthomas

Findings of the eyelid string of pearls and a hoarse cry during the first years of life (due to vocal cord infiltration) is characteristic of Lipoid Proteinosis (AKA Urbach-Wiethe disease or Hyalinosis cutis et mucosae). It is an autosomal recessive condition with mutations in the extracellular matrix protein 1 gene. Other findings include calcifications of the temporal lobe and hippocampus, hairloss, atrophic scars and waxy papules on the face, verrucous nodules and a thick tongue. The other conditions could be considered on the differential for Lipoid Proteinosis, but do not have the findings described above

245) A 12 year-old boy with pits on his palms and lateral fingers may have:

A. Secondary syphilis

B. A corynebacteria infection

C. A hereditary keratoderma

D. Arsenic exposure

E. An inherited cancer syndromeCorrect Choice

Basal cell nevus syndrome is an autosomal dominant disease caused by mutations in the PTCH1 gene. Clinically, patients may have numerous basal cell carcinomas, palmoplantar pits, jaw cysts, frontal bossing, bifid ribs, calcification of falx cerebri, medulloblastoma, ovarian fibromas and fibrosarcomas

89

Page 44: ETAS_MCQ_03 b genodermatoses

246) A patient with colon cancer is diagnosed with Muir-Torre syndrome. Which of the following cutaneous lesions might the patient have?

A. Tricholemmomas

B. Basal cell carcinomas

C. Seborrheic keratoses

D. Keratoacanthomas Correct Choice

E. Arsenical keratoses

Muir-Torre syndrome is an autosomal dominant disease caused by mutations in MSH2 and MSH1, DNA mismatch repair genes. Clinically, patients have multiple sebaceous tumors (adenomas are most common), keratoacanthomas, and are at risk for adenocarcinoma of the colon

247) A patient with multiple sebaceous adenomas should be screened with which of the following examinations?

A. Colonoscopy Correct Choice

B. Retinal examination

C. Renal ultrasound

D. MRI of the spine

E. Laryngoscopy

Muir-Torre syndrome is an autosomal dominant disorder caused by the HMSH2 and MLH1 DNA mismatch repair genes. Clinically, there are numerous sebaceous adenomas, epitheliomas and carcinomas and multiple keratoacanthomas associated with indolent colon and other visceral adenocarcinomas. Patients and first-degree relatives should be screened by colonoscopy as colonic adenocarcinomas may precede the development of cutaneous tumors

248) “Coast of Maine” café au lait macules are characteristic of which condition

A. Hypomelanosis of Ito

B. McCune-Albright syndrome Correct Choice

C. Tuberous sclerosis

D. Carney complex

E. Gaucher’s syndrome

90

Page 45: ETAS_MCQ_03 b genodermatoses

McCune-Albright syndrome is a sporadic condition caused by somatic mutations in the Gs subunit of adenylate cyclase. Key clinical features include “coast of Maine” café au lait macules, polyostotic fibrous dysplasia, and precocious puberty.

249) Patients with junctional epidermolysis bullosa have been found to have mutations in:

A. All of the answers are correctCorrect Choice

B. laminin 5

C. collagen 17

D. bullous pemphigoid antigen 2

E. BP180

All of the answers are correct. Laminin 5 is a protein integral in the adhesion of the dermis to the epidermis. Also involved in junctional epidermolysis bullosa is bullous pemphigoid antigen 2, collagen 17 and BP180, which are synonymous for the same structure

250) Which of the following eye findings is caused by the rupture of Bruch's membrane?

A. Angioid streaksCorrect Choice

B. Blue sclerae

C. Keratoconus

D. Ruptured globe

E. Retinal detachment

The rupture of Bruch's membrane causes angioid streaks in pseudoxanthoma elasticum. Bruch's membrane is the innermost layer of choroid with a central layer of elastic fibers. The other findings are found in Ehlers-Danlos syndrome and are not related to Bruch's membrane

251) A 5 month old girl presents with failure to thrive. She has had life-long atopic dermatitis treated with topical hydrocortisone cream and has persisent hypernatremia. On your exam, she has generalized erythema and scaling of her body and trichorrhexis invaginata on examination of hairs from her eyebrows. Which syndrome is she most likely to have?

A. Netherton SyndromeCorrect Choice

B. Omenn Syndrome

C. Severe atopic dermatitis

D. Wiskott-Aldrich Syndrome

91

Page 46: ETAS_MCQ_03 b genodermatoses

E. Leiner syndrome

Netherton syndrome is caused by a mutation in the SPINK5 gene, encoding LEKT1. This is a serine protease inhibitor which is important in downregulating inflammation. Early presentation is with failure to thrive, generalized erythema/scale, hypernatremia, and sparse hair with the characteristic finding of trichorrhexis invaginata. Pili torti and trichorrhexis nodosa also can be seen. Eyebrow hair is most commonly affected. Omenn syndrome is an autosomal recessive form of severe combined immunodeficiency (SCID) with findings of failure to thrive, erythroderma, scaling, chronic diarrhea, lymphadenopathy, and hepatosplenomegaly. Leiner syndrome can present with failure to thrive, immunodeficiency and seborrheic dermatitis. Wiskott-Aldrich syndrome is an x-linked recessive condition with mutations of the WAS gene. Presentation includes atopic dermatitis, thrombocytopenia, recurrent bacterial infection, lymphoreticular malignancy with non-Hodgkin's lymphoma being the most common, and increased IgA, D and E. With the characteristic hair changes, atopic dermatitis alone is not the most likely diagnosis

252) Patients with hemochromatosis are at increased risk for which of the following?

A. Polyarthritis

B. Yersenia infections

C. Vibrio vulnificus infections

D. Generalized metallic-grey hyperpigmentation

E. All of these options are correctCorrect Choice

Patients with hemochromatosis have increased intestinal iron absorption leading to systemic iron overload. Signs inclued a generalized metallic-grey hyperpigmentation, koilonychia, alopecia (especially pubic/axillary hair) cardiac failure/arrythmias/heart block, hepatomegaly with crrhosis, diabetes (bronze diabetes), polyarthritis with chondrocalcinosis and are susceptible to Vibrio vulnificus and Yersinia infections

253) The diagnostic test for chronic granulomatous disease is:

A. Bone marrow biopsy

B. Histamine skin test

C. Potassium hydroxide

D. Nitroblue tetrazolium reduction assayCorrect Choice

E. Dimethylglyoxime test

Chronic granulomatous disease is characterized by a defect in the ability to kill catalase positive organisms within phagocytic leukocytes. This results from a neutrophilic defect in the cytochrome found in the NADPH oxidative pathway responsible for a respiratory burst. The nitroblue tetrazolium (NBT) reduction assay demonstrates the leukocyes’ ability to reduce the dye and produce a blue color change. Patients with chronic granulomatous disease are unable to redue the dye

92

Page 47: ETAS_MCQ_03 b genodermatoses

254) A 50 year man presents with generalized metallic-grey hyperpigmentation. His past medical history includes diabetes, hepatomegaly and arrythmias. Laboratory tests should include:

A. Iron levelsCorrect Choice

B. Arsenic levels

C. Copper levels

D. Lead levels

E. Cyanide levels

Hemochromatosis is an autosomal recessive disease resulting in increased intestinal iron absorption and iron deposition in a variety of organs. Clinical features include generalized metallic-grey hyperpigmentation, koilonychia, sparse or absent hair, hepatomegaly, cardiac failure/arrhythmias, insulin-dependent diabetes, hypogonadism and polyarthritis

255) Which of the following conditions is worsened by ingestion of lithium?

A. Darier’s DiseaseCorrect Choice

B. Hereditary lymphedema (Nonne-Milroy disease)

C. Hailey-Hailey Disease

D. Haim-Munk syndrome

E. Epidermolytic hyperkeratosis

Patients with Darier’s disease should not be treated with lithium due to its worsening or in some cases unmasking the disease. The mechanism for this is not known. Lithium treatment does not worsen the other listed conditions

256) Which of the following is caused by a defect in keratins 4 & 13?

A. Epidermolysis bullosa simplex

B. Epidermolysis bullosa simplex with myotonic dystrophy

C. Junctional EB with pyloric atresia

D. White sponge nevusCorrect Choice

E. Clouston’s syndrome

White sponge nevus is caused by a defect in keratins 4 & 13.The remaining entities have the corresponding defects:EB simplex—keratins 5 & 14

93

Page 48: ETAS_MCQ_03 b genodermatoses

EB simplex with myotonic dystrophy—plectinJunctional EB with pyloric atresia—Integrin ?-6, ?4Clouston’s syndrome (hidrotic ectodermal dysplasia)—connexin 30.

257) The main cause of death in patients with dyskeratosis congenita is which of the following?

A. Leukemia

B. Pancytopenia Correct Choice

C. Oral squamous cell carcinoma

D. Atherosclerotic heart disease

E. Renal cell carcinoma

Dyskeratosis congenita is usually inherited in an X-recessive fashion due to mutations in the dyskerin gene, which is involved in ribosomal RNA synthesis. The less common autosomal dominant form is caused by mutations in the telomerase gene. Clinically, there is reticulated pigmentation of skin, poikiloderma, alopecia, nail atrophy, premalignant oral leukoplakia, and Fanconi-type pancytopenia resulting in early death

258) Which of the following is NOT a characteristic skin finding in patients with Down Syndrome?

A. Elastosis perforans serpiginosa

B. Syringomas

C. Flat nipples

D. Single palmar crease

E. Small tongueCorrect Choice

Down syndrome is caused by nondisjunction and results in trisomy 21. Clinical features include single palmar crease, flat nipples, increased nuchal skin folds, syringomas, elastosis perforans serpiginosa, xerosis, epicanthic folds of eyes, protruding scrotal tongue and fissured thickened lips.

259) Which vascular disorder is characterized by facial vascular malformation and ipsilateral intracranial and retinal arteriovenous malformations(AVMs)?

A. Encephalotrigeminal angiomatosis

B. Von Lohuizen’s disease

C. PHACES

D. Bonnet Dechaune Blanc syndromeCorrect Choice

94

Page 49: ETAS_MCQ_03 b genodermatoses

E. Sturge-Weber syndome

Bonnet Bechaune Blanc syndome, also know as Wyburn-Mason syndrome, is characterized by a facial vascular malformation and ipsilateral intracranial and retinal AVMs. Encephalotrigeminal angiomatosis is another name for Sturge-Weber. Von Lohuizen's disease is another name for cutis marmorata telangiectatica congenita

260) The syndrome characterized by generalized mild hyperkeratosis, erythematous keratotic plaques, palmoplantar keratoderma, non-progressive sensorineural deafness, progressive bilateral keratitis with secondary blindness is:

A. Vohwinkel syndrome

B. CHILD syndrome

C. KID syndromeCorrect Choice

D. Refsum syndrome

E. Erythrokeratoderma variabilis

KID syndrome is described above. It is an autosomal dominant mutation in connexin 26. Vohwinkel syndrome is also a connexin 26 mutation, but is characterized by diffuse honeycombed palmoplantar keratoderma, pseudoainhum, starfish-shaped keratotic plaques over joints and deafness. Erythrokeratoderma variabilis is an autosomal dominant mutation in connexin 31 and 30.3 characterized by erythematous migratory patches, fixed hyperkeratotic plaques and a palmoplantar keratoderma. CHILD syndrome is an X-linked dominant mutation condition due to a mutation in NAD(P)H Steroid dehydrogenas-like protein, lethal in males. Unilateral ichthyosiform erythroderma, limb/visceral hypoplasias are characteristic. Refsum syndrome is an autosomal recessive condition with a mutation in phytanoyl coenzyme A hydroxylase characterized by mild ichthyosis, cerebellar ataxia, peripheral neuropathy, retinitis pigmentosa (salt & pepper) and deafness

261) The most common cardiovascular defect in patients with Noonan syndrome is:

A. Ventricular septal defect

B. Aortic stenosis

C. Atrial septal defect

D. Enlarged aorta

E. Pulmonic valve stenosisCorrect Choice

Noonan’s syndrome is also known as cardiofaciocutaneous syndrome. Patients have short stature, ptosis, hypertelorism, low-set ears, thick lips and curly hair. Pulmonic valve stenosis is the most common cardiovascular defect, with atrial septal defects also common

262) What nail change is seen in patients with Mal de Meleda Syndrome?

95

Page 50: ETAS_MCQ_03 b genodermatoses

A. leukonychia

B. pterygium

C. KoilonychiaCorrect Choice

D. Longitudinal ridging

E. Onycholysis

Mal de Meleda is an autosomal recessive disease characterized by transgedient malodorous PPK, hyperhidrosis, keratotic plaques at knees and elbows, subungual hyperkeratosis, and koilonychia. The gene defect is SLURP 1

263) Which of the following laboratory test might prove useful in the diagnosis of Fabry disease:

A. Stool guaiac

B. Bleeding time

C. Fasting lipids

D. Urinary sediment exam with polarizing light microscopyCorrect Choice

E. Complete blood count with differential

Patients with Fabry disease have a defect in the alpha-galactosidase A enzyme, leading to an accumulation of glycosphingolipids in all tissues. Although patients are at increased risk for myocardial infrctions and strokes, the serum lipid levels are normal. Ischemic events occur as a result of glycosphingolipid accumulation in endothelial cells leading to swelling. In the brain, strokes occur from direct vessel occlusion or stretching and distention of branches of dolichoectatic parent vessels. Deposits in the kidneys leads to progressive renal failure with urine exam exhibiting proteinuria and birefringent lipid globules (“maltese crosses”) seen with polarizing light microscopy

264) The x-linked recessive type of dyskeratosis congenita is:

A. PTEN

B. dyskerinCorrect Choice

C. CDKN2A

D. Menin

E. TERC

The dyskerin gene, whose product is involved in ribosomal RNA synthesis, is mutated in X-linked recessive dyskeratosis congenita. TERC is linked with autosomal domininant transmission of the syndrome. CDKN2A is involved in familial dysplastic nevi/melanoma syndrome, PTEN in Cowden syndrome and Menin in MEN type I

96

Page 51: ETAS_MCQ_03 b genodermatoses

265) Which of the following conditions is inherited in an X-linked recessive manner?

A. Sjogren-Larsson syndrome

B. Ichthyosis vulgaris

C. Wiskott-Aldrich SyndromeCorrect Choice

D. Epidermolysis bullosa simplex

E. Netherton’s Syndrome

Wiskott-Aldrich Syndrome is inherited in an X-linked recessive manner. Epidermolysis bullosa simplex and ichthyosis vulgaris are inherited in an autosomal dominant (AD) manner. Sjogren-Larsson and Netherton’s syndrome are inherited in an autosomal recessive manner

266) What is this syndrome which is histologically characterized by widely dispersed granular material amidst normal fibers?

A. Focal Dermal Hypoplasia

B. Pseudoxanthoma ElasticumCorrect Choice

C. Buschke-Ollendorf Syndrome

D. Lipoid Proteinosis

E. Ehlers Danlos Syndrome

Pseudoxanthoma elasticum is genodermatosis characterized by redundant skin, angioid streaks, yellow papules on the mucous membranes and bleeding from gastric artery. On histology, readily apparent denerative changes of the elastic fibers are prominent, even without special stains

267) The most common neoplasm seen in Maffucci Syndrome is:

A. EnchondromasCorrect Choice

B. Angiosarcomas

C. chondrosarcoma

D. Lymphangiosarcomas

E. Osteosarcomas

Maffucci syndrome comprises of superficial and deep venous malformations, enchondromas, and short stature. Enchondromas are the most common neoplasm, while chondrosarcomas are the most common malignancies

97

Page 52: ETAS_MCQ_03 b genodermatoses

268) Which of the following syndromes is X-linked dominant?

A. X-linked icthyosis

B. anhidrotic ectodermal dysplasia

C. dyskeratosis congenita

D. Menkes kinky hair syndrome

E. orofaciodigital syndrome 1Correct Choice

Orofaciodigital sydrome 1 is an X-linked dominantly inherited disorder caused by a defect in the CXORF5 gene. The rest of the above conditions are inherited in an x-linked recessive pattern.

269) Which malignancy is seen in approximately 15-20% of people with the disease characterized by a defect in a parathyroid hormone receptor protein?

A. osteosarcoma

B. chondrosarcomaCorrect Choice

C. angiosarcoma

D. rhabdomyosarcoma

E. epitheliod sarcoma

Approximately 15-20% of patients with Maffucci's syndrome will develop chondrosarcoma. Maffucci's syndrome is due to a defect in a parathyroid hormone receptor protein

270) Medulloblastoma is seen in which syndrome?

A. Basal cell nevus syndromeCorrect Choice

B. Gardner’s syndrome

C. Multiple endocrine neoplasia 2b

D. Muir-Torre syndrome

E. Neurofibromatosis Type 1

Basal cell nevus syndrome is an autosomal dominant syndrome caused by a mutation in PTC gene, which acts in the Sonic hedgehog pathway. Cutaneous manifestations of this genodermatosis include basal cell carcinomas, palmoplantar pits, epidermoids cysts. Other findings include odotogenic cysts, frontal bossing, bifid ribs, calcification of the falx cerebri and medulloblastomas

98

Page 53: ETAS_MCQ_03 b genodermatoses

271) A 2 year old girl presents with sunken eyes, large ears, microcephaly and a photodistributed eruption on her face. Eye exam reveals “salt and pepper” retina. The gene responsible for this syndrome codes for a:

A. Transcription factor

B. DNA helicaseCorrect Choice

C. Lysosomal protease

D. Surface glycoprotein

E. Mismatch repair gene

The patient described has Cockayne syndrome, an autosomal recessive disorder believed to be due to a mutation in either DNA helicase or defective nucleotide excision repair. UV irradiated cells have decreased DNA and RNA synthesis and increased chromosomal breaks. Clinical features include cachectic dwarfism with microcephaly, thin nose, large ears, photosensitive eruption, cataracts, salt & pepper retina, and diffuse demyelination

272) Which of the following findings is characteristic of a mutation in lamin A?

A. Lipoatrophic sclerodermoid skin

B. All of the answers are correctCorrect Choice

C. Craniomegaly with small face

D. Severe premature atherosclerosis with early death

E. Alopecia

A mutation in Lamin A causes Progeria (Hutchinson-Gilford syndrome). Other findings include nail atrophy and muscle/bone wasting. Presentation is in the first or second year of life. An increased urine hyaluronic acid can be helpful in diagnosis

273) Which disease is found more commonly in mothers of patients with chronic granulomatous disease?

A. Erythema nodosum

B. Sarcoidosis

C. Discoid lupus erythematousCorrect Choice

D. Wegener's disease

99

Page 54: ETAS_MCQ_03 b genodermatoses

E. Churg-Straus disease

Female carriers of chronic granulomatous disease have an increase incidence of discoid lupus, infections and apthous stomatitis

274) Osteopathia striata is seen in which of the following disorders?

A. Aplasia cutis congenita

B. Scleroderma

C. Focal dermal hypoplasia Correct Choice

D. Albright’s syndrome

E. Gaucher’s disease

Focal dermal hypoplasia (Goltz syndrome) is an X-linked dominant disorder that is lethal in males. There is linear atrophy following Blaschko’s lines with areas of fat herniation with underlying osteopathia striata, which is radiologically characterized by linear bony hyperdensity. Other features include mucocutaneous papillomas and pits, alopecia, nail dystrophy, tooth abnormalities, and colobomas

275) Xeroderma pigmentosum (XP) variant is different than classic XP in which of the following way?

A. Increased chromosomal breakage and sister chromatid exchanges

B. Defective DNA nucleotide excision repair of the global genome

C. Defective DNA nucleotide excision repair of actively transcribing genes

D. Defective post-replication repairCorrect Choice

E. Low IgM

XP variant is DNA nucleotide excision repair proficient, but the defect is in post replication repair of DNA. Increased chromosomal breakage and sister chromatid exchanges is found in Bloom’s syndrome, an autosomal recessive syndrome caused by a defect in BLM gene, whose product functions as a helicase. Clinical findings include: Telangiectasias, short stature, malar erythema, recurrent infection, increased frequency of leukemia and lymphoma, normal intelligence. Defective DNA nucleotide excision repair of actively transcribing genes is a feature of Cockayne’s syndrome, an autosomal recessive syndrome with clinical findings including: Cachexia, short stature, pigmentary retinal degeneration, progressive deafness and no increase in neoplasms. Xeroderma pigmentosum has seven different complementation groups (A-G), each associated with a different form of impairment of DNA nucleotide excision repair

276) A child presents with sparse, short hair and sensorineural deafness. On microscopic examination of the hair, pili torti is noted. Which of the following syndromes is the most likely diagnosis?

100

Page 55: ETAS_MCQ_03 b genodermatoses

A. None of the options are correct

B. Menkes kinky hair syndrome

C. Bjornstad syndromeCorrect Choice

D. Argininosuccinic aciduria

E. Trichothiodystrophy

Bjornstad syndrome is the most likely diagnosis. This rare syndrome (~25 cases) is autosomal recessive. Findings are of deafness and pili torti. The most common hair finding in Menkes syndrome is pili torti, but it is not associated with hearing loss. Argininosuccinic aciduria is associated with trichorrhexis nodosa and has no associated hearing loss

277) Which genetic defect could explain these cutaneous findings in addition to abnormal immunoglobulin levels, recurrent respiratory infections, hypogonadism, and an increased risk of leukemia and lymphoma?

A. Adenosine deaminase

B. ERCC6

C. WAS gene

D. RecQL3Correct Choice

E. NADPH oxidase

Bloom's syndrome is an autosomal recessive disorder caused by mutations in the RecQL3 gene encoding a DNA helicase. Clinically, individuals with Bloom's syndrome have a photodistributed erythema with telangectasia on the malar eminences. The may also have decreased IgM and IgA levels, hypogonadism, and an increased risk for leukemia and lymphoma

278) A patient with multiple lentigines and blue nevi may also have:

A. mental retardation

B. deafness

C. atrial myxomaCorrect Choice

D. pulmonary valve stenosis

E. GI malignancy

This patient may have a constellation of features associated with NAME syndrome, otherwise known as Carney complex or LAMB syndrome. This condition is inherited in an autosomal dominant pattern and is due to a defect in the PRKAR1A gene. This condition is characterized by the following features: blue Nevi, Atrial myxomas, cutaneous Myxomas, and Ephelides. In addition, testicular

101

Page 56: ETAS_MCQ_03 b genodermatoses

tumors are seen as well as sexual precocity. Finally, patients may have endocrine abnormalities including pigmented nodular adrenocorticoal disease and Cushing syndrome as well as pituitary adenomas. Deafness, pulmonary stenosis, GI malignancies, and mental retardation are not features of this condition.

(Spitz's Genodermatoses. 1996, p70-71)(Abdelmalek, N. J Am Acad Dermatol 2002;46:161-183).

279) Ehlers-Danlos Syndrome with congenital adrenal hyperplasia is caused by mutations affecting which of the following?

A. Fibronectin

B. Lysyl oxidase

C. Lysyl hydroxylase

D. Tenascin-X Correct Choice

E. Collagen 5

Ehlers-Danlos Syndrome type with congenital adrenal hyperplasia is caused by mutations in tenascin-X

280) Which of the following diseases is seen only in females?

A. Hypomelanosis of Ito

B. Carney complex

C. Piebaldism

D. Griscelli syndrome

E. Incontinentia pigmenti Correct Choice

Incontinentia pigmenti (Bloch-Sulzberger syndrome) is an X-linked dominant disease that is lethal in males. There are four stages: vesicular, verrucous, hyperpigmented and hypopigmented. Also seen are peg and conical teeth, eye abnormalities, CNS defects, and alopecia. This condition is caused by mutations in the NEMO gene

281) Which of the following is caused by a defect in cathepsin C?

A. Muir-Torre

B. Nail-Patella syndrome

C. Rubenstein-Taybi

102

Page 57: ETAS_MCQ_03 b genodermatoses

D. Papillon-LefevreCorrect Choice

E. Bullous Icthyosis of Siemens

Papillon-Lefevre is a palmoplantar keratoderma caused by a mutation in chromosome 11q14, leading to a defect in Cathepsin C, a lysosomal enzyme. Clinical manifestations of papillon lefevre include sharply demarcated palmoplantar keratoderma with extension to dorsal surface (transgrediens), spare hair, periodontitis, and pyoderma

282) Which of the following syndromes is characterized by follicular atrophoderma, hypohidrosis, hypotrichosis and multiple basal cell carcinomas?

A. Bazex syndromeCorrect Choice

B. Rombo syndrome

C. Incontinentia Pigmenti

D. Rasmusen syndrome

E. Gorlin syndrome

Bazex syndrome has the findings of follicular atrophoderma, hypohidrosis, hypotrichosis and multiple basal cell carcinomas (BCC). Rombo syndrome is associated with BCC and hypotrichosis, but not the other listed findings. The atrophoderma in Rombo syndrome is vermicular, not follicular. Rasmusen syndrome is not associated with BCC

283) Patients with x-linked icthyosis are more prone to getting which two malignancies?

A. Renal cell and ALL

B. Testicular and AML

C. Pancreatic and acute myelogenous leukemia (AML)

D. Testicular and ALLCorrect Choice

E. Pancreatic and acute lymphocytic leukemia (ALL)

Patients with x-linked icthyosis have a 20% chance of having cryptorchidism and are more prone to both testicular cancer and acute lymphocytic leukemia

284) Odontogenic cysts and palmoplantar pits are seen in:

A. Bloom’s Syndrome

B. Refsum syndrome

103

Page 58: ETAS_MCQ_03 b genodermatoses

C. Gorlin SyndromeCorrect Choice

D. Gardner’s syndrome

E. Goltz Syndrome

Gorlin syndrome (Basal Cell Nevus Syndrome)is an autosomal recessive disorder due to a defect in the PTCH gene whose function normally inhibits “SMOOTHENED” signaling. Odontogenic cysts and palmoplantar pits are characteristic features, in addition to multiple basal cell carcinomas. Other features include: frontal bossing, kyphoscoliosis, calcification of falx cerebri, hypertelorism, ovarian fibromas and rarely mental retardation

285) Comma shaped corneal opacities are seen in what disease?

A. Refsum Syndrome

B. Proteus syndrome

C. Sjogren-Larson Syndrome

D. X-linked ichthyosisCorrect Choice

E. Pseudoxanthoma elasticum

X-linked ichthyosis is a X-linked recessive disorder secondary to steroid sulfatase deficiency characterized by brown adherent scale. Additional findings include comma-shaped corneal opacities, cryptorchisdism, and failure to progress during labor.

286) Which of the following is caused by a mutation in a gene that leads to defective NF-KB activation?

A. Vohwinkel’s

B. MEN IIa

C. Chediak-Higashi

D. Incontinentia pigmentiCorrect Choice

E. Piebaldism

Incontinential pigmenti is an X-linked dominant disorder caused by a mutation in the NEMO gene located at Xq28. NEMO is an NF-KB modulator. A genetic defect in NEMO leads to defective activation of NF-KB (a transcription factor). MEN IIa is caused by a mutation in the RET proto-oncogene, piebaldism is caused by the C-kit protocogene, chediak-higashi is caused by a mutation in lysosomal tracking, and Vohwinkel’s is caused by a defect in the loricrin gene, which codes for a structural protein

287) Giant lysosomal granules are seen in which disease?

104

Page 59: ETAS_MCQ_03 b genodermatoses

A. Griscelli syndrome

B. Piebaldism

C. Incontinentia pigmenti

D. Carney complex

E. Chediak-Higashi syndrome Correct Choice

Chediak-Higashi syndrome is caused by an autosomal recessive mutation in a lysosomal transport gene (LYST, CHS1). This disorder is characterized by oculocutaneous albinism, ataxia, muscle weakness, and giant lysosomal granules. There is an accelerated phase characterized by lymphohistiocytic infiltration of reticuloendothelial system, pancytopenia and death

105